Sei sulla pagina 1di 15

Math 21B: Winter 2013

Final Exam Solutions


1. [20 pts.] Evaluate the following indefinite integrals:
∫ ∫ ∫
x+1 1 1
(a) dx; (b) dx; (c) dx.
x2 + 2x + 7 x2 + 2x + 5 x2 + 2x − 3

Solution.
• (a) Substitute u = x2 + 2x + 7, du = 2(x + 1) dx:
∫ ∫
x+1 1 du
dx =
x2 + 2x + 7 2 u
1
= ln |u| + C
2
1
= ln(x2 + 2x + 7) + C.
2
• (b) Complete the square in the denominator and use a tangent substi-
tution x + 1 = 2 tan θ, dx = 2 sec2 θ dθ:
∫ ∫
1 1
2
dx = dx
x + 2x + 5 (x + 1)2 + 4

2 sec2 θ
= dθ
4 tan2 θ + 4

1
= dθ
2
1
= θ+C
2 ( )
1 −1 x+1
= tan + C.
2 2

• (c) Use partial fractions:


1 1
=
x2 + 2x − 3 (x − 1)(x + 3)
A B
= +
x−1 x+3
(A + B)x + (3A − B)
=
(x − 1)(x + 3)

1
We therefore want

A + B = 0, 3A − B = 1

which gives A = 1/4, B = −1/4. It follows that


∫ ∫ ( )
1 1 1 1
dx = − dx
x2 + 2x − 3 4 x−1 x+3
1
= (ln |x − 1| − ln |x + 3|) + C
4
1 x − 1
= ln + C.
4 x + 3

2
2. [20 pts.] Evaluate the following definite integrals:
∫ 1 √ ∫ π ∫
10 v √ 2
(a) 3/2 )2
dv; (b) 1 − cos 2x dx; (c) x ln x dx.
0 (1 + v 0 1

Solution.

• (a) Substitute u = 1 + v 3/2 , du = (3/2)v 1/2 dv. The v-limits v = 0,


v = 1 become u = 1, u = 2, respectively, so
∫ 1 √ ∫ [ ]2
10 v 20 2 du 20 1 10
3/2 2
dv = 2
= − = .
0 (1 + v ) 3 1 u 3 u 1 3

• (b) The double angle formula gives 1 − cos 2x = 2 sin2 x, so


∫ π √ ∫ π √ √

1 − cos 2x dx = 2 sin x dx = 2 [− cos x]π0 = 2 2.
0 0

Note that sin x ≥ 0 for 0 ≤ x ≤ π, so 1 − cos 2x (the positive square
root) is sin x, not − sin x.

• (c) Integrate by parts to remove the logarithm:


1 1
u = ln x, dv = x dx; du = dx, v = x2
x 2
This gives
∫ 2 [ ]2 ∫ 2
1 2 1 2 1
x ln x dx = x ln x − x · dx
1 2 1 1 2 x
[ ]2 ∫ 2
1 1
= x2 ln x − x dx
2 1 2 1
[ ]2
1 2 1 2
= x ln x − x
2 4 1
3
= 2 ln 2 − .
4

3
8

4
y

R
1

0
−1 −0.5 0 0.5 1 1.5 2
x

Figure 1: The region R.

3. [20 pts.] Let A be the area of the region R enclosed between the curves
y = ex and y = e−x for 0 ≤ x ≤ 1.
(a) Sketch the graphs and shade the region R.
(b) Write down a definite integral for the area A.
(c) Evaluate the integral in (b) and find A.

Solution.
• (a) See Figure 1.
• (b) We have ∫ 1 ( )
A= ex − e−x dx.
0

• (c) Evaluating the integral, we get


[ ]1 1
A = ex + e−x 0 = e + − 2.
e

4
1

0.8

0.6
y

0.4

0.2

0
0 0.2 0.4 0.6 0.8 1
x

Figure 2: The area between the line and the curve is rotated about the y-axis.

4. [20 pts.] Suppose that the area between the curve y = x2/3 and the line
3(y − 1) = 2(x − 1) with 0 ≤ x ≤ 1 is rotated about the y-axis (see Figure 2).
Find the volume V of the resulting solid of revolution.

Solution.
• For this problem, it’s easiest to split the area into vertical rectangles and
use “shells” with radius x, circumference 2πx, and height the difference
in the y-coordinates:
∫ 1 [ ]
2
V = 2πx (x − 1) + 1 − x 2/3
dx.
0 3

• Simplifying and evaluating the integral, we get


∫ 1( )
2 2 1
V = 2π x + x−x 5/3
dx
0 3 3
[ ]1
2 3 1 2 3 8/3
= 2π x + x − x
9 6 8 0
π
= .
36

5
Figure 3: The reservoir is filled to a depth h with a triangular dam at the
end.

5. [20 pts.] Suppose that a reservoir has a vertical dam in the shape of an
isosceles triangle with base a and depth h (see Figure 3). Find the pressure
force exerted on the dam when the reservoir is full of water. Express your
answer in terms of a, h and the weight density w = ρ0 g of water.

Solution.

• Let z measure the depth from the top of the reservoir.

• The pressure p at depth z is p = wz.

• The cross-sectional length L of the dam varies linearly from L = a at


z = 0 to L = 0 at z = h, so L = a(h − z)/h.

• The total pressure force F on the dam, with cross-sectional element of


area dA = L dz, is
∫ ∫ h ∫ h
a(h − z)
F = p dA = pL dz = wz · dz.
0 0 h

6
• Evaluating this integral, we get
∫ h
wa
F = (hz − z 2 ) dz
h 0
[ ]h
wa 1 2 1 3
= hz − z
h 2 3 0
1
= wah2 .
6

• Note that w has dimensions of Force/Volume and ah2 has dimensions


of Volume, so F has dimensions of Force as it should.

7
6. [20 pts.] Consider the curve with parametric equation

x = t3 , y = t2 + 1 for 1 ≤ t ≤ 2.

(a) Write down a definite integral for the arclength


(b) Evaluate the integral and find the length of the curve.

Solution.

• (a) The arclength L is given by



∫ 2 ( )2 ( )2
dx dy
L= + dt
1 dt dt
∫ 2√
= (3t2 )2 + (2t)2 dt
∫1 2 √
= 9t4 + 4t2 dt
1

• (b) To evaluate the integral, we factor a t2 out of the square root and
substitute u = 9t2 + 4, du = 18t dt:
∫ 2 √
L= t 9t2 + 4 dt
1
∫ 40
1 √
= u du
18 13
[ ]40
1 2 3/2
= u
18 3 13
1 [ ]
= (40)3/2 − (13)3/2 .
27

8
7. [20 pts.] (a) Solve the initial value problem

dy
= y ln y, y(0) = 2.
dt

(b) How does your solution behave as t → ∞?

Solution.

• (a) Separating variables, we get


∫ ∫
dy
= dt = t + C.
y ln y

• To evaluate the integral of the left-hand side, we substitute u = ln y,


du = dy/y, which gives
∫ ∫
dy du
= = ln |u| + C = ln | ln y| + C.
y ln y u

• The solution is given by

ln | ln y| = t + C,

which implies that ln y = Cet , where C is a different arbitrary constant,


or
t
y = eCe .

• The initial condition gives 2 = eC or C = ln 2, so


t
y = e(ln 2)e .

• (b) Although the solution is defined for all −∞ < t < ∞, it grows very
rapidly (at a doubly-exponential rate) as t → ∞. For example,

y(1) ≈ 6.58, y(2) ≈ 167.62, y(3) ≈ 1, 112, 626, y(5) ≈ 4.75×1044 .

9
8. [20 pts.] Partition the interval 1 ≤ x ≤ 3 into two equal subintervals with
endpoints {1, 2, 3}, and let ∫ 3
1
I= dx.
1 x

(a) Write down the Riemann sum for I using this partition and the left
endpoints.
(b) Write down the Riemann sum for I using this partition and the right
endpoints.
(c) Write down the Riemann sum for I using this partition and the midpoints.
(d) Use your previous answers to give numbers m, M such that m ≤ ln 3 ≤
M . Explain why.

Solution.

• (a) The left Riemann sum, using the endpoints {1, 2}, is
1 1 3
L= ·1+ ·1= .
1 2 2

• (b) The right Riemann sum, using the endpoints {2, 3}, is
1 1 5
R= ·1+ ·1= .
2 3 6

• (c) The midpoint Riemann sum, using the points {3/2, 5/2}, is
1 1 16
M= ·1+ ·1= .
3/2 5/2 15

• (d) The function 1/x is decreasing, so the left Riemann sum is an upper
Riemann sum and the right Riemann sum is a lower Riemann sum. It
follows that ∫ 3
5 1 3
≤ dx ≤ .
6 1 x 2
Evaluating the integral, we get that
5 3
≤ ln 3 ≤ .
6 2

10
Remark. The midpoint Riemann sum M gives a better approximation of
ln 3 than L or R, but we don’t immediately know if it’s an upper or a lower
bound. In fact,
ln 3 ≈ 1.0986, M ≈ 1.0667.
Also note that the midpoint Riemann sum is different from the result T of
the trapezoidal rule, which gives the average of the left and right Riemann
sums:
L+R 7
T = = ≈ 1.1667.
2 6

11
1.5

0.5

0
y

−0.5

−1

−1.5
0 2 4 6 8 10 12
x

Figure 4: The green line y = f (x) is


∫ xthe graph of f , and the blue line is the
graph of its indefinite integral y = √π f (t) dt.

9. [20 pts.] Let


1 1
f (x) = sin(x2 ) + cos(x2 ), F (x) = cos(x2 ).
2x2 x
(a) Show that F ′ (x) = −2f (x).
(b) Define the following improper integral as a limit of Riemann integrals:
∫ ∞

f (x) dx. (1)
π

(c) Show that the improper integral (1) converges, and evaluate it.
(d) How does f (x) behave as x → ∞? Are you surprised that the improper
integral (1) converges?

12
Solution.

• (a) By the product and chain rules,


( )
′ 1 1
F (x) = · (− sin x ) · (2x) + − 2 · cos x2
2
x x
( )
1
= −2 sin x2 + 2 cos x2
2x
= −2f (x).

• (b) We write the improper integral as a limit of Riemann integrals over


a finite interval:
∫ ∞ ∫ b

f (x) dx = lim √ f (x) dx.
π b→∞ π

The √
Riemann integrals are well-defined since f (x) is continuous for
x ≥ π.

• (c) By the Fundamental Theorem of Calculus,


∫ b ∫
1 b ′

f (x) dx = − √ F (x) dx
π 2 π
1
= − [F (x)]b√π
2( )
1 cos b2 1
=− +√ ,
2 b π

where we have used cos π = −1.

• We have
cos b2
lim = 0,
b→∞ b
since | cos b2 | ≤ 1 and the denominator b becomes arbitrarily large.
Thus, ∫ ∞ ( )
1 cos b2 1 1

f (x) dx = − lim +√ =− √ ,
π 2 b→∞ b π 2 π

and the integral converges to −1/(2 π).

13
(d) The function f (x) does not approach 0 as x → ∞, but oscillates
roughly between −1 and 1. It may seem surprising that the improper
integral exists even though the integrand does not approach 0. The
explanation is that the oscillations in the integrand become more rapid
as x gets larger. This increases the cancellation between the positive
and negative contributions to the integral and allows the integral to
have a definite limit (see Figure 4) as x → ∞.

14

∫10. [20 pts.] (a) Find an
∫ equation that
∫ expresses sec3 x dx in terms of
sec x dx. Hint. Write sec3 x dx = sec x · sec2 x dx, integrate by parts,
and use an appropriate trigonometric identity.

(b) Evaluate sec3 x dx.

Solution.
• (a) We set

u = sec x, dv = sec2 x dx; du = sec x tan x dx, v = tan x.

Then an integration by parts gives


∫ ∫
sec x dx = sec x · sec2 x dx
3


= sec x tan x − sec x tan2 x dx.

• Using the Pythagorean identity, We have


( )
sec x tan2 x = sec x sec2 x − 1 = sec3 x − sec x,

so ∫ ∫ ∫
sec x dx = sec x tan x −
3 3
sec x dx + sec x dx.

• Solving this equation for sec3 x dx, we get
∫ ∫
3 1 1
sec x dx = sec x tan x + sec x dx.
2 2

• (b) Using the standard integral for the secant, we obtain



1 1
sec3 x dx = sec x tan x + ln | sec x + tan x| + C.
2 2

Remark. More generally, a similar argument to the one in (a) gives the
following reduction formula for powers of secants:
∫ ∫
n 1 n−2 n−2
sec x dx = sec x tan x + secn−2 x dx.
n−1 n−1

15

Potrebbero piacerti anche